You are on page 1of 18

CHAPTER

10

Conservation of Angular Momentum

1* True or false: (a ) If two vectors are parallel, their cross product must be zero. (b ) When a disk rotates about its symmetry axis, is along the axis. (c) The torque exerted by a force is always perpendicular to the force. (a ) True (b ) True (c) True 2 Two vectors A and B have equal magnitude. Their cross product has the greatest magnitude if A and B are (a ) parallel. (b ) equal. (c) perpendicular. (d ) antiparallel. (e) at an angle of 45o to each other. (c ) A force of magnitude F is applied horizontally in the negative x direction to the rim of a disk of radius R as shown in Figure 10-29. Write F and r in terms of the unit vectors i, j, and k , and compute the torque produced by the force about the origin at the center of the disk. F = -F i; r = R j; = r F = FR j -i = FR i j = FR k . Compute the torque about the origin for the force F = -mg j acting on a particle at r = x i + y j, and show that this torque is independent of the y coordinate. Use Equs. 10-1 and 10-7 = -mgx i j - mgy j j = -mgx k Find A B for (a ) A = 4 i and B = 6 i + 6 j, (b ) A = 4 i and B = 6 i + 6 k , and (c) A = 2 i + 3 j and B = -3 i + 2 j. Use Equ. 10-7; Note that i i = j j = k k = 0 (a ) A B = 24 i j = 24 k . (b ) A B = 24 i k = -24 j. (c) A B = 4 i j - 9 j i = 13 k . 6 7 Under what conditions is the magnitude of A B equal to AB? o o A B = AB sin = AB = AB cos if sin = cos or tan = 1; = 45 or = 135 . A particle moves in a circle of radius r with an angular velocity . (a ) Show that its velocity is v = r. (b ) Show that its centripetal acceleration is a c = v = ( r). (a) Let r be in the xy plane. Then if points in the positive z direction, i.e., = k , the particles velocity is in the j direction when r = r i (see Figure) and has the magnitude r. Thus, v = r = r j.

5*

Chapter 10

Conservation of Angular Momentum

(b ) a = d v/dt = (d /dt) r + (d r/dt) = (d /dt) r + v = a t + ( r) = a t + a c, where a t and a c are the tangential and centripetal accelerations, respectively.

If A = 4 i, Bz = 0, B = 5, and A B = 12 k , determine B. 12 k = 4By i j = 4By k ; By = 3 B =Bx i + By j (Bz = 0); write A B 2 2 2 Bx + By = B ; solve for Bx Bx = 4 ; B = 4 i + 3 j If A = 3 j, A B = 9 i, and AB = 12, find B. Let B = Bx i + By j + Bz k ; write AB and find By Write A B and determine Bx and By

9*

AB = 3By = 12; By = 4 9 i = 3Bx j i + 3Bz j k = -3Bx k + 3Bz i; Bx = 0, Bz = 3 B=4 j+3 k

10 What is the angle between a particles linear momentum p and its angular momentum L? From Equ. 10-8 it follows that L and p are mutually perpendicular; i.e., the angle is 90o. 11 A particle of mass m is moving with speed v along a line that passes through point P. What is the angular momentum of the particle about point P? (a ) mv (b ) zero (c) It changes sign as the particle passes through point P. (d ) It depends on the distance of point P from the origin of coordinates. (b ) 12 A particle travels in a circular path. (a ) If its linear momentum p is doubled, how is its angular momentum affected? (b ) If the radius of the circle is doubled but the speed is unchanged, how is the angula r momentum of the particle affected? (a ) L is doubled. (b ) L is doubled. 13* A particle moves along a straight line at constant speed. How does its angular momentum about any point vary over time? L = mr p is constant. 14 A particle moving at constant velocity has zero angular momentum about a particular point. Show that the particle either has passed through that point or will pass through it. o L = 0, therefore r v = 0. Since neither r nor v is zero, sin = 0, where is the angle between r and v. = 0 or 180o. 15 A 2-kg particle moves at a constant speed of 3.5 m/s around a circle of radius 4 m. (a ) What is its angular momentum about the center of the circle? (b ) What is its moment of inertia about an axis through the center of the circle and perpendicular to the plane of the motion? (c) What is the angular speed of the particle? (a ) L = r p ; L = rmv L = (4 2 3.5) kg.m2/s = 28 kg.m2/s (b ) I = mr2 (c) = L/I I = (2 42) kg.m2 = 32 kg.m2 = 0.875 rad/s

Chapter 10

Conservation of Angular Momentum

16 A 2-kg particle moves at constant speed of 4.5 m/s along a straight line. (a ) What is the magnitude of its angular momentum about a point 6 m from the line? (b ) Describe qualitatively how its angular speed about that point varies with time. (a ) L = rmv sin = (6 2 4.5) kg.m2/s = 54 kg.m2/s. (b ) increases as the particle approaches the point, then decreases as it recedes. 17* A particle is traveling with a constant velocity v along a line that is a distance b from the origin O (Figure 10-30). Let dA be the area swept out by the position vector from O to the particle in time dt. Show that dA/dt is constant in time and equal to 1/2L/m, where L is the angular momentum of the particle about the origin. The area at t = t1 is A1 = 1/2br1 cos 1 = 1/2bx1, where 1 is the angle between r1 and v and x1 is the component of r1 in the direction of v. At t = t1 + dt, A = A1 + dA = 1/2b (x + dx) = 1/2b (x + v dt). Thus, dA/dt = 1/2bv = constant. Note that r sin = b ; consequently , 1/2bv = 1/2L/m. 18 A 15-g coin of diameter 1.5 cm is spinning at 10 rev/s about a vertical diameter at a fixed point on a tabletop. (a ) What is the angular momentum of the coin about its center of mass? (b ) What is its angular momentum about a point on the table 10 cm from the coin? If the coin spins about a vertical diameter at 10 rev/s while its center of mass travels in a straight line across the tabletop at 5 cm/s, (c) what is the angular momentum of the coin about a point on the line of motion? (d ) What is the angular momentum of the coin about a point 10 cm from the line of motion? (There are two answers to this question. Explain why and give both.) (a ) L = Lspin = Icmspin; I = mr2/4 (Problem 9-44) L = (0.015 0.00752/4)(20 ) kg.m2/s (b ) L = Lorbit + Lspin; Lorbit = 0 (c) Lorbit = 0 (Problem 10-14) (d ) Lorbit = mvr = 7.5 10 kg.m2/s
-5

= 1.33 10-5 kg.m2/s L = 1.33 10-5 kg.m2/s L = 1.33 10-5 kg.m2/s L = 8.83 10-5 kg.m2/s; L = -6.17 10-5 kg.m2/s

19 Two particles of masses m1 and m2 are located at r1 and r2 relative to some origin O as in Figure 10-31. They exert equal and opposite forces on each other. Calculate the resultant torque exerted by these internal forces about the origin O and show that it is zero if the forces F1 and F2 lie along the line joining the particles. = i = r1 F1 + r2 F2 = (r1 - r2) F1 since F2 = -F1. But r1 - r2 points along -F1 so (r1 - r2) F1= 0. 20 True or false: (a ) The rate of change of a systems angular momentum is always parallel to the net external torque. (b ) If the net torque on a body is zero, the angular momentum must be zero. (a ) True (see Equ. 10-13) (b ) False 21* A 1.8-kg particle moves in a circle of radius 3.4 m. The magnitude of its angular momentum relative to the center of the circle depends on time according to L = (4 N.m)t. (a ) Find the magnitude of the torque acting on the particle. (b ) Find the angular speed of the particle as a function of time. (a ) = dL/dt = 4 N.m 2 2 2 (b ) = t; = /I; I = mr ; = t/mr = (4/1.8 3.4 )t rad/s = 0.192t rad/s 22 A uniform cylinder of mass 90 kg and radius 0.4 m is mounted so that it turns without friction on its fixed

Chapter 10

Conservation of Angular Momentum

symmetry axis. It is rotated by a drive belt that wraps around its perimeter and exerts a constant torque. At time t = 0, its angular velocity is zero. At time t = 25 s, its angular velocity is 500 rev/min. (a ) What is its angular momentum at t = 25 s? (b ) At what rate is the angular momentum increasing? (c) What is the torque acting on the cylinder? (d ) What is the magnitude of the force acting on the rim of the cylinder? (a ) L = I; I = 1/2mr2 L = (1/2 90 0.42 500 2 /60) kg.m2/s = 377 kg.m2/s (b ) dL/dt = I d/dt = I = constant dL/dt = (377/25) kg.m2/s2 = 15.1 kg.m2/s2 (c) = dL/dt = 15.1 N.m (d ) = Fr F = (15.1/0.4) N = 3.77 N 23 In Figure 10-32, the incline is frictionless and the string passes through the center of mass of each block. The pulley has a moment of inertia I and a radius r. (a ) Find the net torque acting on the system (the two masses, string, and pulley) about the center of the pulley. (b ) Write an expression for the total angular momentum of the system about the center of the pulley when the masses are moving with a speed v. (c) Find the acceleration of the masses from your results for parts (a ) and (b ) by setting the net torque equal to the rate of change of the angula r momentum of the system. (a ) The only external forces acting on the system are m1g and m2g sin . (The normal reaction force of the plane on m2 is balanced by the component of m2g normal to the plane.) Thus net = rg(m2 sin - m1), where we have taken clockwise to be positive to be consistent with a positive upward velocity of m1 as indicated in the figure. (b ) L = I + m1vr + m2vr = vr(I/r2 + m1+ m2). (c) = dL/dt = ar(I/r2 + m1 + m2) = rg(m2 sin - m1); a = (m2 sin - m1)g /(I/r2 + m1 + m2). 24 From her elevated DJ booth at a dance club, Caroline is lowering a 2-kg speaker using a 0.6-kg disk of radius 8 cm as a pulley (Figure 10-33). The speaker wire runs straight up from the speaker, over the pulley, and then horizontally across the table. She attaches the wire to the 4-kg amplifier on her tabletop, and then turns to get the other speaker. The table, however, is nearly frictionless, and the whole system begins to move when she lets go. (a ) What is the net torque about the center of the pulle y? (b ) What is the total angular momentum of the system 3.5 s after release? (c) What is the angular momentum of the pulley at this time? (d ) What is the ratio of the angular momentum of each piece of equipment to the angular momentum of the pulley? (a ) Proceed as in Problem 23 = (2 9.81 0.08) N.m = 1.57 N.m (b ) L = dt L = (1.57 3.5) kg.m2/s = 5.5 kg.m2/s (b) From Problem 23, find a ( = 0) and v; then a = [(2 9.81)/(1/2 0.6 + 6)] m/s2 = 3.11 m/s2; find Ip and . v = 10.9 m/s; I = (0.3 0.082) kg.m2 = 0.00192 kg.m2; 2 = v/r = 136 rad/s; Lp = Ip = 0.262 kg.m /s (d ) Find L(m1) and L(m2); m1 = speaker, m2 = amplifier Determine the ratios L(m1) = (2 10.9 0.08) kg.m2/s = 1.74 kg.m2/s; L(m2) = 3.48 kg.m2/s L(m1)/Lp = 6.64; L(m2)/Lp = 13.3

25* Work Problem 24 for the case in which the coefficient of friction between the table and the 4-kg amplifier is 0.25.

Chapter 10

Conservation of Angular Momentum

(a ) In this case, =m1gr - km2gr = (1.57 - 0.25 4 9.81 0.08) = 0.785 N.m. Now proceed as in Problem 24. (b ) L = 2.75 kg.m2/s. (c) Lp = 0.131 kg.m2/s. (d ) The ratios are the same as in Problem 24. 26 Figure 10-34 shows the rear view of a spaceship that is rotating about its longitudinal axis at 6 rev/min. The occupants wish to stop this rotation. They have small jets mounted tangentially, at a distance R = 3 m from the axis, as indicated, and can eject 10 g/s of gas from each jet with a nozzle velocity of 800 m/s. For how long must they turn on these jets to stop the rotation? The rotational inertia of the ship around its axis (assumed to be constant) is 4000 kgm2. t = L/ = I/; = 2 FR; F = (10-2 800) N t = [(12 /60) 4000/(2 10-2 800 3)] s = 52.4 s

27 True or false: If the net torque on a rotating system is zero, the angular velocity of the system cannot change. False; the moment of inertia could change. 28 Folk wisdom says that a cat always lands on its feet. If a cat starts falling with its feet up, how can it land on its feet without violating the law of conservation of angular momentum? If the cat stretches its feet sideways, air friction provides torque to turn the animal about. 29* If the angular momentum of a system is constant, which of the following statements must be true? (a ) No torque acts on any part of the system. (b ) A constant torque acts on each part of the system. (c) Zero net torque acts on each part of the system. (d ) A constant external torque acts on the system. (e) Zero net torque acts on the system. (e ) 30 Two identical cylindrical disks have a common axis. Initially, one of the disks is spinning. When the two disks are brought into contact they stick together. Which of the following statements is true? (a ) The total kinetic energy and the total angular momentum are unchanged from their initial values. (b ) Both the total kinetic energy and the total angular momentum are reduced to half of their original values. (c) The total angular momentum is unchanged, but the total kinetic energy is reduced to half its original value. (d ) The total angular momentum is reduced to half of its original value, but the total kinetic energy is unchanged. (e) The total angular momentum is unchanged, and the total kinetic energy is reduced to one-quarter of its original value. (c ) 31 In Example 10-4, does the force exerted by the merry-go-round on the child do work? No; in each inelastic collision the force of static friction does not act through any distance. 32 Is it easier to crawl radially outward or radially inward on a rotating merry-go-round? Why? It is easier to crawl out; in the rotating reference frame the effective force on the person is outward. 33* A block sliding on a frictionless table is attached to a string that passes through a hole in the table. Initially, the block is sliding with speed v0 in a circle of radius r0. A student under the table pulls slowly on the string. What happens as the block spirals inward? Give supporting arguments for your choice. (a ) Its energy and angular momentum are conserved. (b ) Its angular momentum is conserved, and its energy increases. (c) Its angular momentum is conserved, and its energy decreases. (d ) Its energy is conserved, and its angular momentum increases. (e) Its energy is conserved, and its angular momentum decreases. (b ) = 0, so L is conserved. The student does work, Fs 0, so the energy of the block must increase. 34 A planet moves in an elliptical orbit about the sun with the sun at one focus of the ellipse as in Figure 10-35. (a )

Chapter 10

Conservation of Angular Momentum

What is the torque produced by the gravitational force of attraction of the sun for the planet? (b ) At position A, the planet is a distance r1 from the sun and is moving with a speed v1 perpendicular to the line from the sun to the planet. At position B, it is at distance r2 and is moving with speed v2, again perpendicular to the line from the sun to the planet. What is the ratio of v1 to v2 in terms of r1 and r2? (a ) r F = 0 since F acts along the direction of r =0 (b ) = 0 = dL/dt; L is constant At A and B, r v = rv; so v1/v2 = r2/r1 35 Under gravitational collapse (all forces on various pieces are inward toward the center), the radius of a spinning spherical star of uniform density shrinks by a factor of 2, with the resulting increased density remaining uniform throughout as the star shrinks. What will be the ratio of the final angular speed 2 to the initial angular speed 1? (a ) 2 (b ) 0.5 (c) 4 (d ) 0.25 (e) 1.0 (c) Since L is constant, I11 = I22; I R2, so 2/1 = R12/R22 = 4. 36 A man stands on a frictionless platform that is rotating with an angular speed of 1.5 rev/s. His arms are outstretched, and he holds a heavy weight in each hand. The moment of inertia of the man, the extended weights, and the platform is 6 kg.m2. When the man pulls the weights inward toward his body, the moment of inertia decreases to 1.8 kg.m2. (a ) What is the resulting angular speed of the platform? (b ) What is the change in kinetic energy of the system? (c) Where did this increase in energy come from? (a ) dL/dt = 0, so f = iIi/If fin = (1.5 6/1.8) rev/s = 5 rev/s 2 2 2 (b ) Ki=1/2Iiwi ;Kf = Ki(Iff /Iii ) K f = 3.33 K i; K = 2.33K i = 621.8J = Kif /i (c) The energy comes from the internal energy of the man 37* A small blob of putty of mass m falls from the ceiling and lands on the outer rim of a turntable of radius R and moment of inertia I0 that is rotating freely with angular speed i about its vertical fixed symmetry axis. (a ) What is the postcollision angular speed of the turntable plus putty? (b ) After several turns, the blob flies off the edge of the turntable. What is the angular speed of the turntable after the blob flies off? 2 (a ) ext = 0; I0i = (I0 + mR2)f f = i /(1 + mR /I0) (b ) When m flies off, its angular momentum does not change = f

38 Two disks of identical mass but different radii (r and 2r) are spinning on frictionless bearings at the same angular speed 0 but in opposite directions (Figure 10-36). The two disks are brought slowly together. The resulting frictional force between the surfaces eventually brings them to a common angular velocity. What is the magnitude of that final angular velocity in terms of 0? 2 I1 = 4I2; f = (3/5) 0 = 0.30 = 0; Li = Lf; so f = 0(I1 - I2)/(I1 + I2); I R 39 A block of mass m sliding on a frictionless table is attached to a string that passes through a hole in the table. Initially, the block is sliding with speed v0 in a circle of radius r0. Find (a ) the angular momentum of the block, (b ) the kinetic energy of the block, and (c) the tension in the string. A student under the table now pulls slowly on the string.

Chapter 10

Conservation of Angular Momentum

How much work is required to reduce the radius of the circle from r0 to r0/2? (a ) L0 = r0mv0. (b ) K0 = 1/2mv02. (c) T = mv02/r0. (d ) Find If and Kf = Lf2/2If = L02/2If If = mr02/4 = I0/4; so Kf = 4K0 W = Kf - K0 W = 3K0 = (3/2) mv02 40 At the beginning of each term, a physics professor named Dr. Zeus shows the class his expectations of them through a demonstration that he calls Lesson #1. He stands at the center of a turntable that can rotate without friction. He then takes a 2-kg globe of the earth and swings it around his head at the end of a 0.8-m chain. The world revolves around him every 3 s, and the professor and the platform have a moment of inertia of 0.5 kg.m2. (a ) What is the angular speed of the professor? (b ) What is the total kinetic energy of the globe, professor, and platform? We shall consider the 2-kg globe to be a point mass. (a ) = 0; Li = 0 = Lf. Write an expression for Lf. Solve for . (b ) K = 1/2M glvgl2 + Lp2/2Ip

Lf = (2 0.82) (2 /3) + 0.5 = 0 = -5.36 rad/s, direction opposite to that of globe K = [1/2 2 (0.8 2 /3)2 + (0.5 5.36) 2/2 0.5] J = 10 J

41* The suns radius is 6.96 108 m, and it rotates with a period of 25.3 d. Estimate the new period of rotation of the sun if it collapses with no loss of mass to become a neutron star of radius 5 km. 2 2 2 5 2 -9 2 = 1(R1 /R2 ) (see Problem 35); T2 = T1(R2/R1) ; T2 = [25.3(5/6.96 10 ) ] days = 1.31 10 days = 0.11 ms. Note: This assumes that the mass distribution in the sun and neutron star are the same. However, the suns mass is concentrated near its center, whereas the density of the neutron star is nearly constant. The correct period will be substantially greater than 0.11 ms. 42 Arriving at the baggage claim area in a small airport, Alan ( mass m) discovers a large turntable (radius R and moment of inertia I) that is spinning out of control. Not wanting to pass up an opportunity for magnificence, Alan leaps onto the edge of the turntable, which continues to spin freely with an angular speed of 7.5 rad/s. He struggles on his hands and knees to the center, and then rises up into a pose that resembles a hood ornament and spins like a figure skater in finale. Security is notified, but passengers applaud. Assume that mR2 = 2.8I, and that Alan has a moment of inertia of I/10 in his final pose. What is his final angular speed if friction is neglected? = 0; Li = Lf. Li = (1 + 2.8) I 7.5 = (1 + 0.1) If. f = (3.8 7.5/1.1) rad/s = 25.9 rad/s. 43 A 0.2-kg point mass moving on a frictionless horizontal surface is attached to a rubber band whose other end is fixed at point P. The rubber band exerts a force F = bx toward P, where x is the length of the rubber band and b is an unknown coefficient. The mass moves along the dotted line in Figure 10-37. When it passes point A, its velocity is 4 m/s directed as shown. The distance AP is 0.6 m and BP is 1.0 m (a ) Find the velocity of the mass at points B and C. (b ) Find b . (a ) = 0; LA = LB = LC; L = mr2 = mvr vB = vArA/rB = 2.4 m/s; vC = vA = 4 m/s 2 2 2 2 2 2 (b ) Ei = Ef ; E = 1/2mv + 1/2bx 1/2 0.2 4 + 1/2b 0.6 = 1/2 0.2 2.4 + 1/2b 1 Solve for b b = 3.2 N/m

Chapter 10 44

Conservation of Angular Momentum

A 2-g particle moves at a constant speed of 3 mm/s around a circle of radius 4 mm. (a ) Find the magnitude of the _ angular momentum of the particle. (b ) If L = l ( l + 1) h , find the value of l (l + 1) and the approximate value of l. (c) Explain why the quantization of angular momentum is not noticed in macroscopic physics. (a ) L = mvr L = (2 10-3 3 10-3 4 10-3) kg.m2/s _ (b ) l (l + 1) = L2/h 2 (c) One cant tell between l = 2 1026 and l = 2 1026 + 1 = 2.4 10-8 kg.m2/s l (l + 1) =(2.4 10-8/1.05 10-34)2 = 5.22 1052; l = 2.29 1026

45* The z component of the spin of an electron is 1/2h , but the magnitude of the spin vector is What is the angle between the electrons spin angular momentum vector and the z axis? cos = 0.5/(0.75) 1/2 ; = 54.7o.

0.75 h

46 Show that the energy difference between one rotational state and the next higher state is proportional l + 1 (see Equation 10-20a ). E = (l + 1)(l + 2)Er0 -l(l + 1)Er0 = 2(l + 1)Er0. 47 In the HBr molecule, the mass of the bromine nucleus is 80 times that of the hydrogen nucleus (a single proton); consequently, in calculating the rotational motion of the molecule, one may, to a good approximation, assume that the Br nucleus remains stationary as the H atom (mass 1.67 10-27 kg) revolves around it. The separation between the H atom and bromine nucleus is 0.144 nm. Calculate (a ) the moment of inertia of the HBr molecule about the bromine nucleus, and (b ) the rotational energies for l = 1, l = 2, and l = 3. (a ) I = mpr2 I = (1.67 10-27 1.442 10-20) kg.m2 (b ) Use Equ. 10-20b to find Er0 Use Equ. 10-20a to find El for l = 1, 2, 3 = 3.46 10-47 kg.m2 Er0 = 1.61 10-22 J = 1.0 meV E1 = 2 meV, E2 = 6 meV, E3 = 12 meV

48 The equilibrium separation between the nuclei of the nitrogen molecule is 0.11 nm. The mass of each nitrogen nucleus is 14 u, where u = 1.66 10-27 kg. We wish to calculate the energies of the three lowest angular momentum states of the nitrogen molecule. (a ) Approximate the nitrogen molecule as a rigid dumbbell of two equal point masses, and calculate the moment of inertia about its center of mass. (b ) Find the rotational energy levels using the relation El _ = l(l + 1)h 2/2I. (a ) I = 2mNr2, where r = 0.11/2 nm I = [28 1.66 10-27 (0.55) 2 10-20] kg.m2 _ (b ) Find Er0 = h 2/2I and use Equ. 10-20a = 1.41 10-46 kg.m2 Er0 = 3.96 10-23 J = 0.25 meV; El = 0.25l (l + 1) meV

Chapter 10

Conservation of Angular Momentum

49* A 16.0-kg, 2.4-m-long rod is supported on a knife edge at its midpoint. A 3.2-kg ball of clay is dropped from rest from a height of 1.2 m and makes a perfectly inelastic collision with the rod 0.9 m from the point of support (Figure 10-38). Find the angular momentum of the rod-and-clay system immediately after the inelastic collision. Li = Lf ; find L just prior to collision L = mvr = [3.2 (2 9.81 1.2) 1/2 0.9] J.s = 14 J.s 50 Figure 10-39 shows a thin bar of length L and mass M , and a small blob of putty of mass m. The system is supported on a frictionless horizontal surface. The putty moves to the right with velocity v, strikes the bar at a distance d from the center of the bar, and sticks to the bar at the point of contact. Obtain expressions for the velocity of the systems center of mass and for the angular velocity of the system about its center of mass. 1. There are no external forces or torques acting on the system. Therefore vcm,i = vcm,f and Li = Lf. 2. ycm = md/(M+m) below the center of the bar. 3. vcm = mv/(M+m). 4. L (about CM) = mv(d - ycm) = mMvd/(M+m). 5. Determine Icm, using the parallel axis theorem: Icm = ML2/12 + Mycm2 + m(d-ycm)2 =
M L 2 ( M 2 + m2 ) d 2 + . 12 M +m

L mM vd 6. Use L = I: = = . 2 I (M L /12)(M + m) + ( M 2 + m2 ) d 2

51 In Problem 50, replace the blob of putty with a small hard sphere of negligible size that collides elastically with the bar. Find d such that the sphere is at rest after the collision. 1. As in Problem 50, ext = Fext = 0. 2. Let v and V be the final velocities of m and M , respectively. Use conservation of linear momentum and angular momentum: mv = mv + MV. (1) 2. mvd = mvd + ML2/12. (2) 3. Set v = 0; V = mv/M. (3) 4. Use energy conservation: mv2 = M(V)2 + ML2 2/12. (4) 5. Use (2) and (3) in (4) and simplify: 1 = m/M + (12m/M )(d 2/L2).
M - m . 12 m

6. Solve for d : d = L

52 Figure 10-40 shows a uniform rod of length L and mass M pivoted at the top. The rod, which is initially at rest, is struck by a particle of mass m at a point d = 0.8L below the pivot. Assume that the collision is perfectly inelastic. What must be the speed v of the particle so that the maximum angle between the rod and the vertical is 90o? Use conservation of L about pivot to find immediately after collision, then use energy conservation to determine v of mass m before collision for an arbitrary angle . Then set = 90o.

Chapter 10

Conservation of Angular Momentum

1. Conservation of angular momentum: 0.8Lmv = I = (ML2/3 + 0.64L2m). = (0.8Lmv)/(ML2/3 + 0.64mL2). 0.32(Lmv )2 2. Conservation of energy: [MgL/2 + mg(0.8L)](1 - cos ) = 1/2I2 = . ML2 /3 + 0.64 mL 2
(0.5M + 0.8m)( ML2 /3 + 0.64 mL 2 )g(1 - cos ) . 0.32 Lm 2

3. Solve for v: v =

4. For = 90o, 1 - cos = 1. 53* If, for the system of Problem 52, L = 1.2 m, M = 0.8 kg, and m = 0.3 kg, and the maximum angle between the rod and the vertical is 60o, find the speed of the particle before impact. Substitute numerical values in result for v of Problem 52, using (1 - cos 60o) = 0.5. v = 7.75 m/s. 54 A projectile of mass mp is traveling at a constant velocity v0 toward a stationary disk of mass M and radius R that is free to rotate about a pivot through its axis O (Figure 10-41). Before impact, the projectile is traveling along a line displaced a distance b below the axis. The projectile strikes the disk and sticks to point B. Treat the projectile as a point mass. (a ) Before impact, what is the total angular momentum L0 of the projectile and disk about the O axis? (b ) What is the angular speed of the disk and projectile system just after the impact? (c) What is the kinetic energy of the disk and projectile system after impact? (d ) How much mechanical energy is lost in this collision? (a ) L0 = mpv0b . (b ) L about the pivot is conserved; I = 1/2MR2 + mpb 2; = L/I = 2mpv0b /(MR2 + mpb 2). (c) Kf = L2/2I = (mpv0b )2/(MR2 + 2mpb 2).

1 mp v2 0 2 (d ) Energy loss is Ki - Kf. Ki = 1/2mpv0 ; E = Ki - Kf = . 2 1 - 2 m p b / MR 2


2

55 A uniform rod of length L1 and mass M = 0.75 kg is supported by a hinge at one end and is free to rotate in the vertical plane (Figure 10-42). The rod is released from rest in the position shown. A particle of mass m = 0.5 kg is supported by a thin string of length L2 from the hinge. The particle sticks to the rod on contact. What should be the ratio L2/L1 so that max = 60o after the collision? 1. Find of rod at impact using energy conservation: MgL1/2 = 1/2(ML12/3) 2; = (3g /L1)1/2. 2. Use conservation of L to find , where is angular speed after impact: = (ML12/3)/(ML12/3 + mL22). 3. Use energy conservation: (3g /L1)(ML12/3)2/(ML12/3 + mL22) = (ML1/2 + mL2)g . 4. Simplify above, using = m/M and = L2/L1: 3 2 3 + 2 + = 2; (4/3) 3 + (2/3) 2 + (2/3) - 2 = 0. 5. Solve the cubic equation for : = 0.88. 56 Returning to Figure 10-42, this time set L1 = 1.2 m, M = 2.0 kg, and L2 = 0.8 m. After the inelastic collision, o max = 37 . Find m. How much energy is dissipated in this inelastic collision? Follow steps 1 and 2 of Problem 55 to obtain an expression for using appropriate numerical values. This gives = 4.75/(0.96 + 0.64m) rad/s. Now use energy conservation, keeping in mind that (1 - cos 37o) = 0.2; Thus, 2 2 solve 1/2 4.75 /(0.96 + 0.64m) J = (2 0.6 + 0.8m) 0.2 9.81 J. Simplify to m + 3.01m - 9.02 = 0 and for m: m = 1.85 kg. E = Ui - Uf ; Ui = MgL1/2 = 11.8 J; Uf = 0.2( MgL1/2 + mgL2) = 5.26 J;

Chapter 10

Conservation of Angular Momentum

E = 6.54 J. 57* Suppose that in Figure 10-42, m = 0.4 kg, M = 0.75 kg , L1 = 1.2 m, and L2 = 0.8 m. What minimum initial angular velocity must be imparted to the rod so that the system will revolve completely about the hinge following the inelastic collision? How much energy is then dissipated in the inelastic collision? Let i and f be the angular velocities of the rod immediately before and immediately after the inelastic collision with the mass m. Let 0 be the initial angular velocity of the rod. We proceed as follows: 1. We apply energy conservation to determine f. 2. Next we apply conservation of angular momentum to determine i. 3. Next, we again apply energy conservation to determine 0. 4. Finally, we find the energies of the system immediately before and immediately after the collision and, thereby, the energy loss. 1. Set K immediately after collision equal to potential energy after 180o rotation. 2 2 2 1/2(ML1 /3 + mL2 )f = MgL1 + 2mgL2; evaluate f : f = 7.0 rad/s. 2. (ML12/3)i = (ML12/3 + mL22)f ; solve for and evaluate i; i = [(0.96 + 0.256)/0.96] f = 8.87 rad/s. 3. 1/2(ML12/3)i2 = 1/2(ML12/3)02 + MgL1/2; 02 = i2 - 3g /L1; 0 = 7.355 rad/s. 4. Ei = 1/2(ML12/3)i2 = 40.95 J; Ef = MgL1 + 2mgL2 = 29.82 J. E = 11.13 J. 58 Repeat Problem 56 if the collision between the rod and particle is elastic. Let v be the speed of m immediately after collision and let i and f be the angular speed of the rod immediately before and immediately after the collision. 1. Use energy conservation at collision MgL1/2 - 0.2MgL1/2 = 1/2mv2; mv2 = 0.8MgL1 = 18.86 J 2 2 2. Use conservation of energy before the collision 1/2(ML1 /3) i = MgL1/2; i = (3g /L1)1/2 = 4.95 rad/s 2 2 3. Use conservation of energy of rod after collision 1/2(ML1 /3) f = 0.2MgL1/2; f = (0.6g /L1)1/2 = 2.215 rad/s 4. Use conservation of angular momentum ML12i /3 = ML12f /3 + mvL2; mv = 3.82 kg.m/s 5. From the results of 1. and 4. find v and m v = (18.86/3.82) m/s = 5.75 m/s; m = 0.665 kg 6. The collision is elastic; E = 0 59 True or false: (a ) Nutation and precession are the same phenomenon. (b ) The direction of precession is the direction of the net torque. (c) When the gyroscope is not spinning, = d L/dt does not hold. (a ) False (b ) True (c) False 60 The angular momentum vector for a spinning wheel lies along its axle and is pointed east. To make this vector point south, it is necessary to exert a force on the east end of the axle in which direction? (a ) Up (b ) Down (c) North (d ) South (e) East (b ) 61* A man is walking north carrying a suitcase that contains a spinning gyroscope mounted on an axle attached to the front and back of the case. The angular velocity of the gyroscope points north. The man now begins to turn to walk east. As a result, the front end of the suitcase will (a ) resist his attempt to turn and will try to remain pointed north. (b ) fight his attempt to turn and will pull to the west. (c) rise upward. (d ) dip downward. (e) cause no effect whatsoever. (d ) 62 The angular momentum of the propeller of a small airplane points forward. (a ) As the plane takes off, the nose lifts up and the airplane tends to veer to one side. To which side does it veer and why? (b ) If the plane is flying

Chapter 10

Conservation of Angular Momentum

horizontally and suddenly turns to the right, does the nose of the plane tend to move up or down? Why? (a ) It veers to the right; this is the direction of the torque associated with the lifting of the nose. (b ) In turning to the right, the torque points down. The nose will tend to move down. 63 A car is powered by the energy stored in a single flywheel with an angular momentum L. Discuss the problems that would arise for various orientations of L and various maneuvers of the car. For example, what would happen if L points vertically upward and the car travels over a hilltop or through a valley? What would happen if L points forward or to one side and the car attempts to turn to the left or right? In each case that you examine, consider the direction of the torque exerted on the car by the road. If L points up and the car travels over a hill or through a valley, the force on the wheels on one side (or the other) will increase and the car will tend to tip. If L points forward and car turns left or right, the front (or rear) of the car will tend to lift. These problems can be averted by having two identical flywheels that rotate on the same shaft in opposite directions. 64 A bicycle wheel of radius 28 cm is mounted at the middle of an axle 50 cm long. The tire and rim weigh 30 N. The wheel is spun at 12 rev/s, and the axle is then placed in a horizontal position with one end resting on a pivot. (a ) What is the angular momentum due to the spinning of the wheel? (Treat the wheel as a hoop.) (b ) What is the angular velocity of precession? (c) How long does it take for the axle to swing through 360o around the pivot? (d ) What is the angular momentum associated with the motion of the center of mass, that is, due to the precession? In what direction is this angular momentum? (a ) L = I; I = MR2 L = (30/9.81) 0.282 (24 ) J.s = 18.1 J.s (b ) p = MgD/L p = (30 0.25/18.1) rad/s = 0.414 rad/s (c) T = 2 /p T = 2 /0.414 s = 15.2 s 2 (d ) Lp = MD p L = 0.079 J.s; up or down, depending on the direction of L
p

65* A uniform disk of mass 2.5 kg and radius 6.4 cm is mounted in the center of a 10-cm axle and spun at 700 rev/min. The axle is then placed in a horizontal position with one end resting on a pivot. The other end is given an initial horizontal velocity such that the precession is smooth with no nutation. (a ) What is the angular velocity of precession? (b ) What is the speed of the center of mass during the precession? (c) What are the magnitude and direction of the acceleration of the center of mass? (d ) What are the vertical and horizontal components of the force exerted by the pivot? 2 (a ) p = MgD/Iss ; Is = 1/2MR2; p = 2gD/R2s p = [2 9.81 0.05/(0.064 700 2 /60)] = 3.27 rad/s (b ) vcm = pD vcm = 3.27 0.05 m/s = 0.163 m/s 2 (c) a cm = Dp a cm = 0.535 m/s2 (d ) Fv = Mg ; Fh = Ma cm Fv = 24.5 N; Fh = 1.34 N 66 An object of mass M is rotating about a fixed axis with angular momentum L. Its moment of inertia about this axis is I. What is its kinetic energy? (a ) IL2/2 (b ) L2/2I (c) ML2/2 (d ) IL2/2M (b ) 67 Explain why a helicopter with just one main rotor has a second smaller rotor mounted on a horizontal axis at the

Chapter 10

Conservation of Angular Momentum

rear as in Figure 10-43. Describe the resultant motion of the helicopter is this rear rotor fails during flight. To prevent the body of the helicopter from rotating. If the rear rotor fails, the body of the helicopter will tend to rotate on the main axis. 68 A woman sits on a spinning piano stool with her arms folded. When she extends her arms out to the side, her kinetic energy (a ) increases. (b ) decreases. (c) remains the same. (b ) L constant but I increases. 69* In tetherball, a ball is attached to a string that is attached to a pole. When the ball is hit, the string wraps around the pole and the ball spirals inward. Neglecting air resistance, what happens as the ball swings around the pole? Give supporting arguments for your choice. (a ) The mechanical energy and angular momentum of the ball are conserved. (b ) The angular momentum of the ball is conserved, but the mechanical energy of the ball increases. (c) The angular momentum of the ball is conserved, and the mechanical energy of the ball decreases. (d ) The mechanical energy of the ball is conserved and the angular momentum of the ball increases. (e) The mechanical energy of the ball is conserved and the angular momentum of the ball decreases. (e) Consider the situation shown in the adjoining figure. The ball rotates counterclockwise. The torque about the center of the pole is clockwise and of magnitude RT, where R is the poles radius and T is the tension. So L must decrease.

70 A uniform rod of mass M and length L lies on a horizontal frictionless table. A piece of putty of mass m = M /4 moves along a line perpendicular to the rod, strikes the rod near its end, and sticks to the rod. Describe qualitatively the subsequent motion of the rod and putty. The center of mass of the rod-and-putty system moves in a straight line, and the system rotates about the center of mass. 71 A particle of mass 3 kg moves with velocity v = 3 m/s i along the line z = 0, y = 5.3 m. (a ) Find the angular momentum L relative to the origin when the particle is at x = 12 m, y = 5.3 m. (b ) A force F = -3 N i is applied to the particle. Find the torque relative to the origin due to this force. (a ) L = r p L = (5.3 3 3) j i = -47.7 k (b ) = r F = (5.3 3) j -i = 15.9 k N.m 72 The position vector of a particle of mass 3 kg is given by r = 4 i + 3t2 j, where r is in meters and t is in seconds. Determine the angular momentum and torque acting on the particle about the origin. L = mr v = mr d r/dt; find d r/dt and L d r/dt = 6t j; L = (3 4 6)t i j J.s = 72t k J.s = d L/dt = 72 k N.m 73* An ice skater starts her pirouette with arms outstretched, rotating at 1.5 rev/s. Estimate her rotational speed (in revolutions per second) when she brings her arms flat against her body. Assume I of body, minus arms, = 1/2 50 0.22 = 1.0 kg.m2. The mass of each arm = 4 kg, and length = 1.0 m.

Chapter 10

Conservation of Angular Momentum

Then Itot(arms out) = 1.0 + 2 4/3 = 3.7 kg.m2 and Itot(arms in) = 1.0 + 2 4 0.22 = 1.32 kg.m2. With L constant, one then finds that = 1.5(3.7/1.32) = 4.2 rev/s. 74 Two ice skaters hold hands and rotate, making one revolution in 2.5 s. Their masses are 55 kg and 85 kg, and they are separated by 1.7 m. Find (a ) the angular momentum of the system about their center of mass, and (b ) the total kinetic energy of the system. (a ) 1. Locate the center of mass from 85 kg mass xcm = 1.7 55/140 m = 0.668 m 2. Determine I and L I = (55 1.0322 + 85 0.6682) kg.m2 = 96.5 kgm2 (b ) K = L /2I
2

L = (96.5 2 /2.5) J.s = 242.5 J.s K = (242.52/193) J = 305 J

75 A 2-kg ball attached to a string of length 1.5 m moves in a horizontal circle as a conical pendulum (Figure 10-44). The string makes an angle = 30o with the vertical. (a ) Show that the angular momentum of the ball about the point of support P has a horizontal component toward the center of the circle as well as a vertical component, and find these components. (b ) Find the magnitude of d L/dt, and show that it equals the magnitude of the torque exerted by gravity about the point of support. Take the coordinate origin at point P. Then r = (1.5 sin 30o)(cos t i + sin t j) - 1.5 cos 30o k ; = k . (a ) 1. Use Newtons laws to determine v; T cos = mg; T sin = mv2/(r sin); v = (rg sin tan )1/2 v = (1.5 9.81/2 v = d r/dt 2. Find L = r p = mr v
3 )1/2 m/s = 2.06 m/s

v = -2.06 sin t i + 2.06 cos t j o o L = 6.18[sin 30 (cos t i + sint j) - cos 30 k ] (-sint i + cos t j) = [(3.09 k + 5.35(sin t j + cos t i)] J.s d L/dt = 5.35 (cos t j - sint i) J; = v/(r sin 30o) = 2.75 rad/s; d L/dt = 14.7 J = 2 9.81 0.75 = 14.7 N.m

(b )1. Find d L/dt; only the horizontal component of L depends on t 2. Find = mgr sin 30o

76 A mass m on a horizontal, frictionless surface is attached to a string that wraps around a vertical cylindrical post so that when it is set into motion it follows a path that spirals inward. (a ) Is the angular momentum pf the mass conserved? (b ) Is the energy of the mass conserved? (c) If the speed of the mass is v0 when the length of the string is r, what is its speed when the unwrapped length has shortened to r/2? See Problem 69. (a ) No (b ) Yes (c) Since K is conserved, v is constant at v0. 77* Figure 10-45 shows a hollow cylindrical tube of mass M , length L, and moment of inertia ML2/10. Inside the cylinder are two masses m, separated a distance l and tied to a central post by a thin string. The system can rotate about a vertical axis through the center of the cylinder. With the system rotating at , the strings holding the masses suddenly break. When the masses reach the end of the cylinder, they stick. Obtain expressions for the final angular velocity and the initial and final energies of the system. Assume that the inside walls of the cylinder are frictionless.

Chapter 10

Conservation of Angular Momentum Ii = (ML2/10 + 2ml 2/4); If = (ML2/10 + 2mL2/4) 2 2 f = [(M + 5ml /L )/(M + 5m)] Ki = (ML2 + 5ml 2) 2/20 Kf = [(ML2 + 5ml 2)2/(ML2 + 5mL2)] 2/20 (1) (2) (3) (4)

1. = 0; Lf = Li; f = (Ii/If). Obtain expressions for Ii and If and f. 2. Ki = 1/2Ii 2; Kf = 1/2Iff2

78 Repeat Problem 77, this time adding friction between the masses and walls of the cylinder. However, the coefficient of friction is not enough to prevent the masses from reaching the ends of the cylinder. Can the final energy of the system be determined without knowing the coefficient of kinetic friction? Again, there is no net external torque. Since the masses, m, come to the same final position, the initial and final configurations are the same as in Problem 77. Therefore, the answers are the same as for Problem 77. 79 Suppose that in Figure 10-45, l = 0.6 m, L = 2.0 m, M = 0.8 kg, and m = 0.4 kg. The system rotates at such that the tension in the string is 108 N just before it breaks. Determine the initial and final angular velocities and initial and final energies of the system. Assume that the inside walls of the cylinder are frictionless. Here we need only to determine . We can then use the expressions derived in Problem 77. m(l /2) 2 = T; = 2T/ml 0 = 30 rad/s. Substitute numerical values into Equ. (2) of Problem 77: f = 10.5 rad/s. Substitute numerical values into Equ. (3) and Equ. (4) of Problem 77: Ki =176.4 J; Kf = 61.74 J. 80 For Problem 77, determine the radial velocity of each mass just before it reaches the end of the cylinder. Until the inelastic collision of the masses m at the ends of the cylinder, both angular momentum and energy are conserved. To determine vr we set K' = Ki, where K' is the kinetic energy of the system just before the masses m reach the end of the cylinder. 1. Write expressions for K and Ki; solve for vr 2. Use the results of Problem 77 and simplify K =
2 1/2Iff

2 1/2(2mvr )
2

Ii 2 - If2 f = 1/2Ii ; v r = 2m
2

vr = ( l/2L) L 2 - l

81* Given the numerical values of Problem 79, suppose the coefficient of friction between the masses and the walls of the cylinder is such that the masses cease sliding 0.2 m from the ends of the cylinder. Determine the initial and final angular velocities of the system and the energy dissipated in friction. As before, angular momentum is conserved. The masses m now come to rest at r = 0.8 m. i = 30 rad/s as before. 1. Use Li = Lf; Find Ii and If Ii = 0.392 kg.m2; If = ML2/10 + 2m 0.82 = 0.832 kg.m2 2. f = i(Ii/If) f = 14.13 rad/s 3. E = Ki - Kf; Ki = 176.4 J (see Problem 79) Kf = 1/2 0.832 14.132 J = 83.1 J; E = 93.3 J 82 Keplers second law states: The radius vector from the sun to a planet sweeps out equal areas in equal times. Show that this law follows directly from the law of conservation of angular momentum and the fact that the force of gravitational attraction between a planet and the sun acts along the line joining the two celestial objects.

Chapter 10

Conservation of Angular Momentum

The drawing shows an elliptical orbit. The triangular element of the area is dA = 2 2 2 1/2r(r d) = 1/2r d . So dA/dt = 1/2r . Since the force acts along r, = 0 and Mr = L is constant. Consequently dA/dt is constant.

83 Figure 10-46 shows a hollow cylinder of length 1.8 m, mass 0.8 kg, and radius 0.2 m that is free to rotate about a vertical axis through its center and perpendicular to the cylinders axis. Inside the cylinder are two thin disks of 0.2 kg each, attached to springs of spring constant k and unstretched lengths 0.4 m. The system is brought to a rotational speed of 8 rad/s with the springs clamped so they do not stretch. The springs are then suddenly unclamped. When the disks have stopped their radial motion due to friction between the disks and the wall, they come to rest 0.6 m from the central axis. What is the angular velocity of the cylinder when the disks have stopped their radial motion? How much energy was dissipated in friction between the disks and cylinder wall? I = Icyl + 2Id; Icyl = 0.8(1.82/12 + 0.22/2) = 0.232; 1. Find initial and final I of system in kg.m2; let x be Id = 0.2(0.22/4 + x2); Ii = 0.3; If = 0.38 the radial distance of each disk f = 8 0.3/0.38 rad/s = 6.316 rad/s 2. f = i(Ii/If) 0.2k = 0.2 0.6 6.3162; k = 23.9 N/m 3. Find the spring constant k ; Ei = 1/2Iii2 = 9.6 J; Ef = 1/2Iff2 + 1/2k x2 = 8.06 J 4. Find initial and final energies of system Wfr = 1.54 J 5. Wfr = Ei - Ef 84 (a ) Assuming the earth to be a homogeneous sphere of radius r and mass m, show that the period T of the earths rotation about its axis is related to its radius by T = (4 m/5L)r2, where L is the angular momentum of the earth due to its rotation. (b ) Suppose that the radius r changes by a very small amount r due to some internal effect such as thermal expansion. Show that the fractional change in the period T is given approximately by T/T = 2r/r. Hint: Use the differentials dr and dT to approximate the changes in these quantities. (c) By how many kilometers would the earth need to expand for the period to change by (1/4) d /y so that leap years would no longer be necessary? (a ) T = 2 /; = L/I; I = 2mr2/5 T = 2 I/L = (4 m/5L)r2 (b ) Find dT/dr; then dT/T dT/dr = 2(4 m/5L)r; dT/T = 2dr/r or T/T = 2r/r 3 (c) T/T = 1/(4 365) r = rT/2T = 1/2(6.37 10 )(1/1460) km = 2.18 km 85* The polar ice caps contain about 2.3 1019 kg of ice. This mass contributes negligibly to the moment of inertia of the earth because it is located at the poles, close to the axis of rotation. Estimate the change in the length of the day that would be expected if the polar ice caps were to melt and the water were distributed uniformly over the surface of the earth. (The moment of inertia of a spherical shell of mass m and radius r is 2mr2/3.) 1. T = 2 I/L (see Problem 84); L is constant dT/T = dI/I or T = TI/I 2 24 2 19 24 -6 2. I = 2M ERE /5; M E = 6 10 kg; I = 2mRE /3 T = (1 d )[(5/3)(2.3 10 /6 20 ) = 6.4 10 d = 0.55 s

Chapter 10

Conservation of Angular Momentum

86 Figure 10-47 shows a hollow cylinder of mass M = 1.2 kg and length L = 1.6 m that is free to rotate about a vertical axis through its center. Inside the cylinder are two disks, each of mass 0.4 kg that are tied to a central post by a thin string and separated by a distance l = 0.8 m. The string breaks if the tension exceeds 100 N. Starting from rest, a torque is applied to the system until the string breaks. Assuming the disks are point masses and the radius of the cylinder is negligible, find the amount of work done up to that instant. Suppose that at that instant, the applied torque is removed, and that the walls of the cylinder are frictionless. Obtain an expression for the angular velocity of the system as a function of x for x < L/2, where x is the distance between each mass and the central post. Icyl = 1.2 1.62/12 = 0.256; 2Im = 2 0.4 x2 = 0.8x2; 1. Find Ii = Icyl + 2Im in kg.m2 Ii = I(x = 0.4) = 0.384; I(x) = 0.256 + 0.8x2 i = (100/0.4 0.4) 1/2 = 25 rad/s 2. Find i when string breaks; mri2 = Tc W = 1/2(0.384 252) = 120 J 3. Work done = K; K = 1/2Iii2 4. Find L; L = Iii = I(x)(x); solve for (x) L = 9.6 J.s; (x) = [9.6/(0.256 + 0.8x2)] rad/s 87 For the system of Problem 86, find the angular velocity of the system just before and just after the point masses pass the ends of the cylinder. Use the result of Problem 86: x = 0.8 m; = [9.6/(0.256 + 0.83)] rad/s = 12.5 rad/s. This is the angular velocity in both instances because the masses leave the cylinder with a tangential velocity of 1/2L so the angular momentum of the system remains constant. 88 Repeat Problem 86 with the radius of the hollow cylinder as 0.4 m and the masses treated as thin disks rather than point masses. 1. To find Icyl and Im we must refer to Table 9.1 and Icyl = M (L2/12 + R2/2) = 0.352; 2Im = 2m(R2/4 + x2) also use the parallel axis theorem. Ii = Icyl + 2Im(x = 0.4) = 0.512 2. i is the same as in Problem 86 i = 25 rad/s 3. W = 1/2Iii2 4. Find L; L = Iii = I(x)(x); solve for (x) W = 1/2 0.512 625 J = 160 J 2 (x) = [12.8/(0.384 + 0.8x )] rad/s

89* Figure 10-48 shows a pulley in the shape of a uniform disk with a heavy rope hanging over it. The circumference of the pulley is 1.2 m and its mass is 2.2 kg. The rope is 8.0 m long and its mass is 4.8 kg. At the instant shown in the figure, the system is at rest and the difference in height of the two ends of the rope is 0.6 m. (a ) What is the angular velocity of the pulley when the difference in height between the two ends of the rope is 7.2 m? (b ) Obtain an expression for the angular momentum of the system as a function of time while neither end of the rope is above the center of the pulley. There is no slippage between rope and pulley. (a) Take y = 0 at center of pulley; write Ui and Uf Ui = -1/2L1i(L1ig ) - 1/2L2i(L2ig ) = -1/2(L1i2 + L2i2)g for the free part of the rope. Let = M r/L Uf = -1/2(L1f2 + L2f2)g ; L1i+L2i = 7.4 m; L2i-L2i = 0.6 m; = 0.6 kg/m. Let L1 and L2 be the lengths of the hanging parts. L1i = 3.4 m, L2i = 4.0 m; also L1f = 0.1 m, L2f = 7.3 m Use conservation of energy: K + U = 0. K = 1/2Ip 2 + 1/2Mv2; v = R; R = (1.2/2 ) m = 0.6/ m

Chapter 10 Write K in terms of . Find U

Conservation of Angular Momentum K = 1/2 1/2 2.2(0.6/ )2 2 + 1/2 8(0.6/ )2 2 = 0.166 2 2 2 2 2 U = Uf - Ui = -1/2(0.1 - 3.4 + 7.3 - 4.0 ) 0.6g = -75.75 J = (75.75/0.166) 1/2 rad/s = 21.36 rad/s U() = -1/2[(L1i - R)2 + (L2i + R)2]g 2 2 2 2 U = -U() + 1/2(L1i + L2i )g = R g 0.166 2 = [(0.6/ )2 0.6 9.81] 2 = 0.215 2; = 1.14 d /dt = 1.14; d / = 1.14 dt ln() = 1.14t; (t) = e1.14t - 1 1.14t (t) = d /dt = 1.14e L = [1/2 2.2(0.6/ )2 + 8.0(0.6/ )2] 1.14e1.14t J.s L = 0.378e1.14t J.s

Solve for (b) Now write U() and U, where is the angle through which the pulley has turned. This will reduce L1 by R and increase L2 by R. Use energy conservation and result for K Recall that = d /dt Integrate Find (t) L = Lp + Lr = (Ip + M rR2); note that the angular momentum of each portion of the rope is the same

You might also like